Minimum number of distinct distances in a planeLower bound for a set of distances between pairs of points in a planePairs of points exactly $1$ unit apart in the planeEmbedding finite (discrete) metric spaces to Eulidean space as isometrically as possibleNumber of possible combinations of the Enigma machine plugboardChoosing k pairs l distance apart from n numbersNearly-unit-distance graph (UDG) densityCombinatoral Geometry with DistancesMinimum number of points to intersect any circle in the regionFinding the number of distinct extrema of polynomial.Set of Integers, find expectation of twin pairs (defined in description)

Unable to disable Microsoft Store in domain environment

Language involving irrational number is not a CFL

Determining multivariate least squares with constraint

What happens if I try to grapple mirror image?

Is there a RAID 0 Equivalent for RAM?

Are inadvertent environmental catastrophes also examples of natural selection?

Can I cause damage to electrical appliances by unplugging them when they are turned on?

Do you waste sorcery points if you try to apply metamagic to a spell from a scroll but fail to cast it?

Has the laser at Magurele, Romania reached a tenth of the Sun's power?

If Captain Marvel (MCU) were to have a child with a human male, would the child be human or Kree?

Should I assume I have passed probation?

The Digit Triangles

Why would five hundred and five be same as one?

Deciphering cause of death?

How can I, as DM, avoid the Conga Line of Death occurring when implementing some form of flanking rule?

Would a primitive species be able to learn English from reading books alone?

Anime with legendary swords made from talismans and a man who could change them with a shattered body

Why do Radio Buttons not fill the entire outer circle?

If the only attacker is removed from combat, is a creature still counted as having attacked this turn?

Why can't the Brexit deadlock in the UK parliament be solved with a plurality vote?

Why does the Persian emissary display a string of crowned skulls?

What is the meaning of "You've never met a graph you didn't like?"

What is this high flying aircraft over Pennsylvania?

Giving feedback to someone without sounding prejudiced



Minimum number of distinct distances in a plane


Lower bound for a set of distances between pairs of points in a planePairs of points exactly $1$ unit apart in the planeEmbedding finite (discrete) metric spaces to Eulidean space as isometrically as possibleNumber of possible combinations of the Enigma machine plugboardChoosing k pairs l distance apart from n numbersNearly-unit-distance graph (UDG) densityCombinatoral Geometry with DistancesMinimum number of points to intersect any circle in the regionFinding the number of distinct extrema of polynomial.Set of Integers, find expectation of twin pairs (defined in description)













0












$begingroup$


Let $U(n)$ denote the maximum possible number of pairs of points in an $n$-point subset of $Bbb R^2$ that are unit distance apart.



Let $g(n)$ be the minimum number of distinct distances determined by $n$ points in $Bbb R^2$, that is:
$$g(n)=textmin_Pleft|textdist(x,y)mid x,yin Pright|.$$



My textbook says "Clearly $g(n)geq nchoose 2 / U(n)$". Why is this so? I have no idea how to even show that this is reasonable. My first thought is that there are clearly $nchoose 2$ pairs and if $U(n)=nchoose 2$, i.e. they are all unit distance apart, then indeed $g(n)=1$. Otherwise as $U(n)$ gets smaller, $g(n)$ should get bigger, which seems okay intuitively. But I don't see how to show this



Actually, a thought after typing this out: Perhaps they are thinking about taking all pairs, and breaking them up into $U(n)$ sized groups, of which would would have $nchoose 2/U(n)$, and one cannot do better than this.










share|cite|improve this question











$endgroup$







  • 1




    $begingroup$
    How does $U(n)$ depend on $n$?
    $endgroup$
    – Fimpellizieri
    Apr 2 '18 at 15:45










  • $begingroup$
    @Fimpellizieri fixed
    $endgroup$
    – user547222
    Apr 2 '18 at 15:46










  • $begingroup$
    Yes, your afterthought is correct. Obviously there are at most $U(n)$ pairs of points that are any given distance apart, by scaling. So there can't be more that $U(n)$ points in any group, so at most $binomn2/U(n)$ groups.
    $endgroup$
    – saulspatz
    Apr 2 '18 at 15:51










  • $begingroup$
    This is pigeon-hole, and in fact I'd even say: Clearly, $g(n)ge 1+nchoose 2/U(n)$ for $nge 2$
    $endgroup$
    – Hagen von Eitzen
    Apr 2 '18 at 15:53
















0












$begingroup$


Let $U(n)$ denote the maximum possible number of pairs of points in an $n$-point subset of $Bbb R^2$ that are unit distance apart.



Let $g(n)$ be the minimum number of distinct distances determined by $n$ points in $Bbb R^2$, that is:
$$g(n)=textmin_Pleft|textdist(x,y)mid x,yin Pright|.$$



My textbook says "Clearly $g(n)geq nchoose 2 / U(n)$". Why is this so? I have no idea how to even show that this is reasonable. My first thought is that there are clearly $nchoose 2$ pairs and if $U(n)=nchoose 2$, i.e. they are all unit distance apart, then indeed $g(n)=1$. Otherwise as $U(n)$ gets smaller, $g(n)$ should get bigger, which seems okay intuitively. But I don't see how to show this



Actually, a thought after typing this out: Perhaps they are thinking about taking all pairs, and breaking them up into $U(n)$ sized groups, of which would would have $nchoose 2/U(n)$, and one cannot do better than this.










share|cite|improve this question











$endgroup$







  • 1




    $begingroup$
    How does $U(n)$ depend on $n$?
    $endgroup$
    – Fimpellizieri
    Apr 2 '18 at 15:45










  • $begingroup$
    @Fimpellizieri fixed
    $endgroup$
    – user547222
    Apr 2 '18 at 15:46










  • $begingroup$
    Yes, your afterthought is correct. Obviously there are at most $U(n)$ pairs of points that are any given distance apart, by scaling. So there can't be more that $U(n)$ points in any group, so at most $binomn2/U(n)$ groups.
    $endgroup$
    – saulspatz
    Apr 2 '18 at 15:51










  • $begingroup$
    This is pigeon-hole, and in fact I'd even say: Clearly, $g(n)ge 1+nchoose 2/U(n)$ for $nge 2$
    $endgroup$
    – Hagen von Eitzen
    Apr 2 '18 at 15:53














0












0








0





$begingroup$


Let $U(n)$ denote the maximum possible number of pairs of points in an $n$-point subset of $Bbb R^2$ that are unit distance apart.



Let $g(n)$ be the minimum number of distinct distances determined by $n$ points in $Bbb R^2$, that is:
$$g(n)=textmin_Pleft|textdist(x,y)mid x,yin Pright|.$$



My textbook says "Clearly $g(n)geq nchoose 2 / U(n)$". Why is this so? I have no idea how to even show that this is reasonable. My first thought is that there are clearly $nchoose 2$ pairs and if $U(n)=nchoose 2$, i.e. they are all unit distance apart, then indeed $g(n)=1$. Otherwise as $U(n)$ gets smaller, $g(n)$ should get bigger, which seems okay intuitively. But I don't see how to show this



Actually, a thought after typing this out: Perhaps they are thinking about taking all pairs, and breaking them up into $U(n)$ sized groups, of which would would have $nchoose 2/U(n)$, and one cannot do better than this.










share|cite|improve this question











$endgroup$




Let $U(n)$ denote the maximum possible number of pairs of points in an $n$-point subset of $Bbb R^2$ that are unit distance apart.



Let $g(n)$ be the minimum number of distinct distances determined by $n$ points in $Bbb R^2$, that is:
$$g(n)=textmin_Pleft|textdist(x,y)mid x,yin Pright|.$$



My textbook says "Clearly $g(n)geq nchoose 2 / U(n)$". Why is this so? I have no idea how to even show that this is reasonable. My first thought is that there are clearly $nchoose 2$ pairs and if $U(n)=nchoose 2$, i.e. they are all unit distance apart, then indeed $g(n)=1$. Otherwise as $U(n)$ gets smaller, $g(n)$ should get bigger, which seems okay intuitively. But I don't see how to show this



Actually, a thought after typing this out: Perhaps they are thinking about taking all pairs, and breaking them up into $U(n)$ sized groups, of which would would have $nchoose 2/U(n)$, and one cannot do better than this.







combinatorics discrete-geometry






share|cite|improve this question















share|cite|improve this question













share|cite|improve this question




share|cite|improve this question








edited Mar 14 at 5:44









Martin Sleziak

44.9k10121274




44.9k10121274










asked Apr 2 '18 at 15:42









user547222user547222

155




155







  • 1




    $begingroup$
    How does $U(n)$ depend on $n$?
    $endgroup$
    – Fimpellizieri
    Apr 2 '18 at 15:45










  • $begingroup$
    @Fimpellizieri fixed
    $endgroup$
    – user547222
    Apr 2 '18 at 15:46










  • $begingroup$
    Yes, your afterthought is correct. Obviously there are at most $U(n)$ pairs of points that are any given distance apart, by scaling. So there can't be more that $U(n)$ points in any group, so at most $binomn2/U(n)$ groups.
    $endgroup$
    – saulspatz
    Apr 2 '18 at 15:51










  • $begingroup$
    This is pigeon-hole, and in fact I'd even say: Clearly, $g(n)ge 1+nchoose 2/U(n)$ for $nge 2$
    $endgroup$
    – Hagen von Eitzen
    Apr 2 '18 at 15:53













  • 1




    $begingroup$
    How does $U(n)$ depend on $n$?
    $endgroup$
    – Fimpellizieri
    Apr 2 '18 at 15:45










  • $begingroup$
    @Fimpellizieri fixed
    $endgroup$
    – user547222
    Apr 2 '18 at 15:46










  • $begingroup$
    Yes, your afterthought is correct. Obviously there are at most $U(n)$ pairs of points that are any given distance apart, by scaling. So there can't be more that $U(n)$ points in any group, so at most $binomn2/U(n)$ groups.
    $endgroup$
    – saulspatz
    Apr 2 '18 at 15:51










  • $begingroup$
    This is pigeon-hole, and in fact I'd even say: Clearly, $g(n)ge 1+nchoose 2/U(n)$ for $nge 2$
    $endgroup$
    – Hagen von Eitzen
    Apr 2 '18 at 15:53








1




1




$begingroup$
How does $U(n)$ depend on $n$?
$endgroup$
– Fimpellizieri
Apr 2 '18 at 15:45




$begingroup$
How does $U(n)$ depend on $n$?
$endgroup$
– Fimpellizieri
Apr 2 '18 at 15:45












$begingroup$
@Fimpellizieri fixed
$endgroup$
– user547222
Apr 2 '18 at 15:46




$begingroup$
@Fimpellizieri fixed
$endgroup$
– user547222
Apr 2 '18 at 15:46












$begingroup$
Yes, your afterthought is correct. Obviously there are at most $U(n)$ pairs of points that are any given distance apart, by scaling. So there can't be more that $U(n)$ points in any group, so at most $binomn2/U(n)$ groups.
$endgroup$
– saulspatz
Apr 2 '18 at 15:51




$begingroup$
Yes, your afterthought is correct. Obviously there are at most $U(n)$ pairs of points that are any given distance apart, by scaling. So there can't be more that $U(n)$ points in any group, so at most $binomn2/U(n)$ groups.
$endgroup$
– saulspatz
Apr 2 '18 at 15:51












$begingroup$
This is pigeon-hole, and in fact I'd even say: Clearly, $g(n)ge 1+nchoose 2/U(n)$ for $nge 2$
$endgroup$
– Hagen von Eitzen
Apr 2 '18 at 15:53





$begingroup$
This is pigeon-hole, and in fact I'd even say: Clearly, $g(n)ge 1+nchoose 2/U(n)$ for $nge 2$
$endgroup$
– Hagen von Eitzen
Apr 2 '18 at 15:53











1 Answer
1






active

oldest

votes


















0












$begingroup$

Given an $n$-set $Ssubset mathbb R^2$, consider



$$mathscr S = (x,y),.$$



Obviously, $|mathscr S| = binomn2$.
Now, define an equivalence relationship $sim$ on $mathscr S$ by



$$ (x_0,y_0) sim (x_1,y_1) iff d(x_0,y_0) = d(x_1,y_1).$$



The relation $sim$ partitions $mathscr S$ into some number $k$ of classes, each of which has most $U(n)$ elements.
Hence, $k geq leftlceilbinomn2/U(n)rightrceil geq binomn2/U(n)$.



Now, we know there are least $g(n)$ classes.
Suppose that we chose $S$ a priori such that $k = g(n)$.
This is possible, for otherwise $g(n)$ would not be a minimum.



With this choice, the inequality follows.
Notice that the inequality depends on $n$, but not on $S$.
Using our freedom to choose $S$ can make it easier to prove the inequality.






share|cite|improve this answer











$endgroup$








  • 1




    $begingroup$
    Really neat, thanks!
    $endgroup$
    – user547222
    Apr 2 '18 at 16:18










  • $begingroup$
    You're welcome! Glad to help.
    $endgroup$
    – Fimpellizieri
    Apr 2 '18 at 16:19










Your Answer





StackExchange.ifUsing("editor", function ()
return StackExchange.using("mathjaxEditing", function ()
StackExchange.MarkdownEditor.creationCallbacks.add(function (editor, postfix)
StackExchange.mathjaxEditing.prepareWmdForMathJax(editor, postfix, [["$", "$"], ["\\(","\\)"]]);
);
);
, "mathjax-editing");

StackExchange.ready(function()
var channelOptions =
tags: "".split(" "),
id: "69"
;
initTagRenderer("".split(" "), "".split(" "), channelOptions);

StackExchange.using("externalEditor", function()
// Have to fire editor after snippets, if snippets enabled
if (StackExchange.settings.snippets.snippetsEnabled)
StackExchange.using("snippets", function()
createEditor();
);

else
createEditor();

);

function createEditor()
StackExchange.prepareEditor(
heartbeatType: 'answer',
autoActivateHeartbeat: false,
convertImagesToLinks: true,
noModals: true,
showLowRepImageUploadWarning: true,
reputationToPostImages: 10,
bindNavPrevention: true,
postfix: "",
imageUploader:
brandingHtml: "Powered by u003ca class="icon-imgur-white" href="https://imgur.com/"u003eu003c/au003e",
contentPolicyHtml: "User contributions licensed under u003ca href="https://creativecommons.org/licenses/by-sa/3.0/"u003ecc by-sa 3.0 with attribution requiredu003c/au003e u003ca href="https://stackoverflow.com/legal/content-policy"u003e(content policy)u003c/au003e",
allowUrls: true
,
noCode: true, onDemand: true,
discardSelector: ".discard-answer"
,immediatelyShowMarkdownHelp:true
);



);













draft saved

draft discarded


















StackExchange.ready(
function ()
StackExchange.openid.initPostLogin('.new-post-login', 'https%3a%2f%2fmath.stackexchange.com%2fquestions%2f2718927%2fminimum-number-of-distinct-distances-in-a-plane%23new-answer', 'question_page');

);

Post as a guest















Required, but never shown

























1 Answer
1






active

oldest

votes








1 Answer
1






active

oldest

votes









active

oldest

votes






active

oldest

votes









0












$begingroup$

Given an $n$-set $Ssubset mathbb R^2$, consider



$$mathscr S = (x,y),.$$



Obviously, $|mathscr S| = binomn2$.
Now, define an equivalence relationship $sim$ on $mathscr S$ by



$$ (x_0,y_0) sim (x_1,y_1) iff d(x_0,y_0) = d(x_1,y_1).$$



The relation $sim$ partitions $mathscr S$ into some number $k$ of classes, each of which has most $U(n)$ elements.
Hence, $k geq leftlceilbinomn2/U(n)rightrceil geq binomn2/U(n)$.



Now, we know there are least $g(n)$ classes.
Suppose that we chose $S$ a priori such that $k = g(n)$.
This is possible, for otherwise $g(n)$ would not be a minimum.



With this choice, the inequality follows.
Notice that the inequality depends on $n$, but not on $S$.
Using our freedom to choose $S$ can make it easier to prove the inequality.






share|cite|improve this answer











$endgroup$








  • 1




    $begingroup$
    Really neat, thanks!
    $endgroup$
    – user547222
    Apr 2 '18 at 16:18










  • $begingroup$
    You're welcome! Glad to help.
    $endgroup$
    – Fimpellizieri
    Apr 2 '18 at 16:19















0












$begingroup$

Given an $n$-set $Ssubset mathbb R^2$, consider



$$mathscr S = (x,y),.$$



Obviously, $|mathscr S| = binomn2$.
Now, define an equivalence relationship $sim$ on $mathscr S$ by



$$ (x_0,y_0) sim (x_1,y_1) iff d(x_0,y_0) = d(x_1,y_1).$$



The relation $sim$ partitions $mathscr S$ into some number $k$ of classes, each of which has most $U(n)$ elements.
Hence, $k geq leftlceilbinomn2/U(n)rightrceil geq binomn2/U(n)$.



Now, we know there are least $g(n)$ classes.
Suppose that we chose $S$ a priori such that $k = g(n)$.
This is possible, for otherwise $g(n)$ would not be a minimum.



With this choice, the inequality follows.
Notice that the inequality depends on $n$, but not on $S$.
Using our freedom to choose $S$ can make it easier to prove the inequality.






share|cite|improve this answer











$endgroup$








  • 1




    $begingroup$
    Really neat, thanks!
    $endgroup$
    – user547222
    Apr 2 '18 at 16:18










  • $begingroup$
    You're welcome! Glad to help.
    $endgroup$
    – Fimpellizieri
    Apr 2 '18 at 16:19













0












0








0





$begingroup$

Given an $n$-set $Ssubset mathbb R^2$, consider



$$mathscr S = (x,y),.$$



Obviously, $|mathscr S| = binomn2$.
Now, define an equivalence relationship $sim$ on $mathscr S$ by



$$ (x_0,y_0) sim (x_1,y_1) iff d(x_0,y_0) = d(x_1,y_1).$$



The relation $sim$ partitions $mathscr S$ into some number $k$ of classes, each of which has most $U(n)$ elements.
Hence, $k geq leftlceilbinomn2/U(n)rightrceil geq binomn2/U(n)$.



Now, we know there are least $g(n)$ classes.
Suppose that we chose $S$ a priori such that $k = g(n)$.
This is possible, for otherwise $g(n)$ would not be a minimum.



With this choice, the inequality follows.
Notice that the inequality depends on $n$, but not on $S$.
Using our freedom to choose $S$ can make it easier to prove the inequality.






share|cite|improve this answer











$endgroup$



Given an $n$-set $Ssubset mathbb R^2$, consider



$$mathscr S = (x,y),.$$



Obviously, $|mathscr S| = binomn2$.
Now, define an equivalence relationship $sim$ on $mathscr S$ by



$$ (x_0,y_0) sim (x_1,y_1) iff d(x_0,y_0) = d(x_1,y_1).$$



The relation $sim$ partitions $mathscr S$ into some number $k$ of classes, each of which has most $U(n)$ elements.
Hence, $k geq leftlceilbinomn2/U(n)rightrceil geq binomn2/U(n)$.



Now, we know there are least $g(n)$ classes.
Suppose that we chose $S$ a priori such that $k = g(n)$.
This is possible, for otherwise $g(n)$ would not be a minimum.



With this choice, the inequality follows.
Notice that the inequality depends on $n$, but not on $S$.
Using our freedom to choose $S$ can make it easier to prove the inequality.







share|cite|improve this answer














share|cite|improve this answer



share|cite|improve this answer








edited Apr 2 '18 at 16:10

























answered Apr 2 '18 at 15:57









FimpellizieriFimpellizieri

17.2k11836




17.2k11836







  • 1




    $begingroup$
    Really neat, thanks!
    $endgroup$
    – user547222
    Apr 2 '18 at 16:18










  • $begingroup$
    You're welcome! Glad to help.
    $endgroup$
    – Fimpellizieri
    Apr 2 '18 at 16:19












  • 1




    $begingroup$
    Really neat, thanks!
    $endgroup$
    – user547222
    Apr 2 '18 at 16:18










  • $begingroup$
    You're welcome! Glad to help.
    $endgroup$
    – Fimpellizieri
    Apr 2 '18 at 16:19







1




1




$begingroup$
Really neat, thanks!
$endgroup$
– user547222
Apr 2 '18 at 16:18




$begingroup$
Really neat, thanks!
$endgroup$
– user547222
Apr 2 '18 at 16:18












$begingroup$
You're welcome! Glad to help.
$endgroup$
– Fimpellizieri
Apr 2 '18 at 16:19




$begingroup$
You're welcome! Glad to help.
$endgroup$
– Fimpellizieri
Apr 2 '18 at 16:19

















draft saved

draft discarded
















































Thanks for contributing an answer to Mathematics Stack Exchange!


  • Please be sure to answer the question. Provide details and share your research!

But avoid


  • Asking for help, clarification, or responding to other answers.

  • Making statements based on opinion; back them up with references or personal experience.

Use MathJax to format equations. MathJax reference.


To learn more, see our tips on writing great answers.




draft saved


draft discarded














StackExchange.ready(
function ()
StackExchange.openid.initPostLogin('.new-post-login', 'https%3a%2f%2fmath.stackexchange.com%2fquestions%2f2718927%2fminimum-number-of-distinct-distances-in-a-plane%23new-answer', 'question_page');

);

Post as a guest















Required, but never shown





















































Required, but never shown














Required, but never shown












Required, but never shown







Required, but never shown

































Required, but never shown














Required, but never shown












Required, but never shown







Required, but never shown







Popular posts from this blog

Lowndes Grove History Architecture References Navigation menu32°48′6″N 79°57′58″W / 32.80167°N 79.96611°W / 32.80167; -79.9661132°48′6″N 79°57′58″W / 32.80167°N 79.96611°W / 32.80167; -79.9661178002500"National Register Information System"Historic houses of South Carolina"Lowndes Grove""+32° 48' 6.00", −79° 57' 58.00""Lowndes Grove, Charleston County (260 St. Margaret St., Charleston)""Lowndes Grove"The Charleston ExpositionIt Happened in South Carolina"Lowndes Grove (House), Saint Margaret Street & Sixth Avenue, Charleston, Charleston County, SC(Photographs)"Plantations of the Carolina Low Countrye

random experiment with two different functions on unit interval Announcing the arrival of Valued Associate #679: Cesar Manara Planned maintenance scheduled April 23, 2019 at 00:00UTC (8:00pm US/Eastern)Random variable and probability space notionsRandom Walk with EdgesFinding functions where the increase over a random interval is Poisson distributedNumber of days until dayCan an observed event in fact be of zero probability?Unit random processmodels of coins and uniform distributionHow to get the number of successes given $n$ trials , probability $P$ and a random variable $X$Absorbing Markov chain in a computer. Is “almost every” turned into always convergence in computer executions?Stopped random walk is not uniformly integrable

How should I support this large drywall patch? Planned maintenance scheduled April 23, 2019 at 00:00UTC (8:00pm US/Eastern) Announcing the arrival of Valued Associate #679: Cesar Manara Unicorn Meta Zoo #1: Why another podcast?How do I cover large gaps in drywall?How do I keep drywall around a patch from crumbling?Can I glue a second layer of drywall?How to patch long strip on drywall?Large drywall patch: how to avoid bulging seams?Drywall Mesh Patch vs. Bulge? To remove or not to remove?How to fix this drywall job?Prep drywall before backsplashWhat's the best way to fix this horrible drywall patch job?Drywall patching using 3M Patch Plus Primer